Question and Answers Forum

All Questions      Topic List

Limits Questions

Previous in All Question      Next in All Question      

Previous in Limits      Next in Limits      

Question Number 18678 by Joel577 last updated on 27/Jul/17

lim_(x→∞)   ((4^(x + 1)  + 2^(x +1)  − 3^(x + 1) )/(4^(x − 1)  + 2^(x − 1 ) + 3^(x + 1)  ))

$$\underset{{x}\rightarrow\infty} {\mathrm{lim}}\:\:\frac{\mathrm{4}^{{x}\:+\:\mathrm{1}} \:+\:\mathrm{2}^{{x}\:+\mathrm{1}} \:−\:\mathrm{3}^{{x}\:+\:\mathrm{1}} }{\mathrm{4}^{{x}\:−\:\mathrm{1}} \:+\:\mathrm{2}^{{x}\:−\:\mathrm{1}\:} +\:\mathrm{3}^{{x}\:+\:\mathrm{1}} \:} \\ $$

Answered by 433 last updated on 27/Jul/17

    ((4^(x+1) +2^(x+1) −3^(x+1) )/(4^(x−1) +2^(x−1) +3^(x+1) ))=(((4^(x+1) /4^(x+1) )+(2^(x+1) /4^(x+1) )−(3^(x+1) /4^(x+1) ))/((4^(x−1) /4^(x+1) )+(2^(x−1) /4^(x+1) )+(3^(x+1) /4^(x+1) )))  =((1+((1/2))^(x+1) −((3/4))^(x+1) )/((1/(16))+((((1/2))^(x+1) )/4)+((3/4))^(x+1) ))→^(x→+∞) ((1+0−0)/((1/(16))+(0/4)+0))=16

$$ \\ $$$$ \\ $$$$\frac{\mathrm{4}^{{x}+\mathrm{1}} +\mathrm{2}^{{x}+\mathrm{1}} −\mathrm{3}^{{x}+\mathrm{1}} }{\mathrm{4}^{{x}−\mathrm{1}} +\mathrm{2}^{{x}−\mathrm{1}} +\mathrm{3}^{{x}+\mathrm{1}} }=\frac{\frac{\mathrm{4}^{{x}+\mathrm{1}} }{\mathrm{4}^{{x}+\mathrm{1}} }+\frac{\mathrm{2}^{{x}+\mathrm{1}} }{\mathrm{4}^{{x}+\mathrm{1}} }−\frac{\mathrm{3}^{{x}+\mathrm{1}} }{\mathrm{4}^{{x}+\mathrm{1}} }}{\frac{\mathrm{4}^{{x}−\mathrm{1}} }{\mathrm{4}^{{x}+\mathrm{1}} }+\frac{\mathrm{2}^{{x}−\mathrm{1}} }{\mathrm{4}^{{x}+\mathrm{1}} }+\frac{\mathrm{3}^{{x}+\mathrm{1}} }{\mathrm{4}^{{x}+\mathrm{1}} }} \\ $$$$=\frac{\mathrm{1}+\left(\frac{\mathrm{1}}{\mathrm{2}}\right)^{{x}+\mathrm{1}} −\left(\frac{\mathrm{3}}{\mathrm{4}}\right)^{{x}+\mathrm{1}} }{\frac{\mathrm{1}}{\mathrm{16}}+\frac{\left(\frac{\mathrm{1}}{\mathrm{2}}\right)^{{x}+\mathrm{1}} }{\mathrm{4}}+\left(\frac{\mathrm{3}}{\mathrm{4}}\right)^{{x}+\mathrm{1}} }\overset{{x}\rightarrow+\infty} {\rightarrow}\frac{\mathrm{1}+\mathrm{0}−\mathrm{0}}{\frac{\mathrm{1}}{\mathrm{16}}+\frac{\mathrm{0}}{\mathrm{4}}+\mathrm{0}}=\mathrm{16} \\ $$

Commented by Joel577 last updated on 28/Jul/17

thank you very much

$${thank}\:{you}\:{very}\:{much} \\ $$

Answered by behi.8.3.4.1.7@gmail.com last updated on 30/Jul/17

l=li_(x→∞) m((4+2((1/2))^x −3((3/4))^x )/((1/4)+(1/2)((1/2))^x +3((3/4))^x ))=(4/(1/4))=16. ■

$${l}={l}\underset{{x}\rightarrow\infty} {{i}m}\frac{\mathrm{4}+\mathrm{2}\left(\frac{\mathrm{1}}{\mathrm{2}}\right)^{{x}} −\mathrm{3}\left(\frac{\mathrm{3}}{\mathrm{4}}\right)^{{x}} }{\frac{\mathrm{1}}{\mathrm{4}}+\frac{\mathrm{1}}{\mathrm{2}}\left(\frac{\mathrm{1}}{\mathrm{2}}\right)^{{x}} +\mathrm{3}\left(\frac{\mathrm{3}}{\mathrm{4}}\right)^{{x}} }=\frac{\mathrm{4}}{\frac{\mathrm{1}}{\mathrm{4}}}=\mathrm{16}.\:\blacksquare \\ $$

Terms of Service

Privacy Policy

Contact: info@tinkutara.com